1971 AHSME Problems/Problem 35

Revision as of 19:49, 8 August 2024 by Thepowerful456 (talk | contribs) (typo fix)

Problem

Each circle in an infinite sequence with decreasing radii is tangent externally to the one following it and to both sides of a given right angle. The ratio of the area of the first circle to the sum of areas of all other circles in the sequence, is

$\textbf{(A) }(4+3\sqrt{2}):4\qquad \textbf{(B) }9\sqrt{2}:2\qquad \textbf{(C) }(16+12\sqrt{2}):1\qquad \\ \textbf{(D) }(2+2\sqrt{2}):1\qquad  \textbf{(E) }(3+2\sqrt{2}):1$

Solution 1

[asy]  import geometry;  point A = origin; point B = dir(135); point C = (0,sqrt(2)); point D = dir(45);  point X = 1/(1+sqrt(2)/2)*(B-C)+C; point Y = 1/(1+sqrt(2)/2)*(D-C)+C;  // Circles draw(circle(A,1)); draw(incircle(triangle(C,X,Y)));  // Segments XY and BD draw(X--Y); draw(B--D);  // Square draw(A--B--C--D--cycle);  // Point Labels dot(A); label("A",A,S); dot(B); label("B",B,W); dot(C); label("C",C,N); dot(D); label("D",D,E); dot(X); label("X",X,NW); dot(Y); label("Y",Y,NE);  [/asy]

$\boxed{\textbf{(C) }(16+12\sqrt2):1}$.

Solution 2 (Informed guess)

Draw a good diagram, and especially make sure that you are drawing a right angle. Then, you will realize that the big circle completely dwarfs all of the other circles. Answer choice (C), $16+12\sqrt2$, is about $16+12\cdot1.4=32.8$, whereas the greatest of the other answer choices is about $6.3$, which is unreasonably small based on our diagram. Thus, we choose answer $\boxed{\textbf{(C) }(16+12\sqrt2):1}$.

See Also

1971 AHSC (ProblemsAnswer KeyResources)
Preceded by
Problem 34
Followed by
Last Problem
1 2 3 4 5 6 7 8 9 10 11 12 13 14 15 16 17 18 19 20 21 22 23 24 25 26 27 28 29 30 31 32 33 34 35
All AHSME Problems and Solutions

The problems on this page are copyrighted by the Mathematical Association of America's American Mathematics Competitions. AMC logo.png